Download as pdf or txt
Download as pdf or txt
You are on page 1of 93

Download All PDF form here :- https://t.

me/pdf4exams

IAS 2018

HISTORY
TEST SERIES
By: PIYUSH KUMAR

TEST: 2

www.iasscore.in
https://t.me/testseries4exams
Download All PDF form here :- https://t.me/pdf4exams

Test - 02

History Test Series 2018

HISTORY
Time Allowed: 3 hr. Max. Marks: 250

SECTION A

1. Identify the following places marked on the map supplied to you and write a short note of
about 30 words on each of them in your Question-cum" Booklet. Locational hints for each of
the places marked on the map are given below seriatim.

E
1. A Jain site

2. A Buddhist educational cente

3. A port site
OR
4. An Ancient capital
SC

5. A Palaeolithic site

6. A historical site

7. A Harappan site

8. An ancient capital
GS

9. A Manufacturing centre in Ancient India

10. A later Vedic site

11. A Mesolithic site

12. A Chalcolithic site

13. A Buddhist centre

14. A trade centre in Ancient India

15. An ancient capital

16. A port site

17. Rock-cave art centre

18. An ancient capital

19. A political and cultural center

20. An ancient town

2. (a) History is a gradual progress of defining the identity of society. Elaborate it with special
reference to Indian history writing.

History [1]

https://t.me/testseries4exams
Download All PDF form here :- https://t.me/pdf4exams

(b) Through the Paleolithic painting draw a sketch of the life-ways of Paleolithic hunters and
gatherers.

(c) Write a short note on the Social organisation and Settlement Pattern of Megalithic culture
in India.

3. (a) To what extent you think that the religious pattern of Indus Valley Civilization continued
even in the later periods?

(b) In architecture there is no real trace of relationship between the brick houses of Harappa
and the temples of Historical age, whether the earliest sculptures of historical times, on the
other hand shows a great likeness to that of Harappa. Elucidate.

(c) "There is an integral connection between urbanism and state societies." Analyse the statement
with respect to urbanism in Indus Valley Civilisation.

SECTION B
4. (a) Upanishads marked a great shift in the philosophical thoughts in the Later Vedic Period.
Analyse.

E
(b) Critically analyse the Panini's work on Sanskrit grammar.
(c) "During the later Vedic Period some remarkable changes could be noticed in the treatment
OR
of vis." Discuss.
5. (a) How far do you agree that the prime concern of Buddhism was society than the religion in
Post Vedic era
(b) "The social stratification was the most important prerequisite for the establishment of state
SC

system, which could be in Ancient India." Critically evaluate.


(c) What were the different trends of the thinking which emerged in the wake of the socio-
religious ferment during Mahajanpada period?
6. (a) "Kautilya's thought which we come to know through Arthsashta, does not have a pragmatic
approach." In the light of given statement critically evaluate Arthsashta.
GS

(b) Chaityas and viharas played major role for the integration and consolidation of Buddhism
during Mauryan period. Explain.
(c) Evaluate the knowledge on medical science of Ancient Indian People.
7. (a) What were the impacts of Persian invasion in India? Is the impact could be noticed in
Ashokan pillars as well?
(b) What were the major nastika philosophies in Ancient India and in what ways it differed
from astika philosophies. Write with special reference to Lokayat sect.
(c) "After the Kushanas, Mahayana Buddhism deviated a lot from the original Buddhism, but
even though it attracted the mass a lot." Critically analyse.
8. (a) Discuss the changing social pattern in post Mauryan and Gupta period with special reference
to verna and jati.
(b) "The Gupta period could be called a golden age for the development in astronomy and
mathematics." Examine.
(c) Over the time it became difficult for the individual to upgrade in the social order but social
mobilization was always evident in Indian society. Critically Examine.


[ 2 ] History

https://t.me/testseries4exams
Download All PDF form here :- https://t.me/pdf4exams

Test - 02

History Test Series 2018

HISTORY
Time Allowed: 3 hr. Max. Marks: 250

Instructions to Candidate

• Please read each of the following instructions carefully before attempting questions

• There are EIGHT questions, printed in ENGLISH.

• Candidate has to attempt FIVE questions in all.

• Question NO- 1 is compulsory and in remaining questions you have to attempt 4

• The number of marks carried by a question/part is indicated against it.

• Answers must be written in the medium authorized in the Admission certificate which must
be stated clearly on the cover of this Question-cum-Answer (QCA) booklet in the space
provided. No marks will be given for answers written in medium other than the authorized
one.

• Word limit in questions, wherever specified, should be adhered to.

• Attempts of questions shall be counted in chronological order. Unless struck off, attempt of
a question shall be counted even if attempted partly.

• Any page or portion of the page left blank in the answer book must be clearly struck off.

Name _______________________________

Mobile No.___________________________
1. Invigilator Signature _______________ Date ________________________________
2. Invigilator Signature _______________ Signature ____________________________

https://t.me/testseries4exams
2
Download All PDF form here :- https://t.me/pdf4exams

REMARKS

Remarks

https://t.me/testseries4exams
3
Download All PDF form here :- https://t.me/pdf4exams
Roll No.____________

SECTION- A

1. Identify the following places marked on the map supplied to you and write a short note
of about 30 words on each of them in your Question-cum" Booklet. Locational hints for
each of the places marked on the map are given below seriatim. (2.5 × 20 = 50 Marks)

1. A Jain site

2. A Buddhist educational cente

3. A port site

4. An Ancient capital

5. A Palaeolithic site

6. A historical site

7. A Harappan site

8. An ancient capital

9. A Manufacturing centre in Ancient India

10. A later Vedic site

11. A Mesolithic site

12. A Chalcolithic site

13. A Buddhist centre

14. A trade centre in Ancient India

15. An ancient capital

16. A port site

17. Rock-cave art centre

18. An ancient capital

19. A political and cultural center

20. An ancient town

Remarks

https://t.me/testseries4exams
4
Download All PDF form here :- https://t.me/pdf4exams

Remarks

https://t.me/testseries4exams
5
Download All PDF form here :- https://t.me/pdf4exams

Remarks

https://t.me/testseries4exams
6
Download All PDF form here :- https://t.me/pdf4exams

Remarks

https://t.me/testseries4exams
7
Download All PDF form here :- https://t.me/pdf4exams

Remarks

https://t.me/testseries4exams
8
Download All PDF form here :- https://t.me/pdf4exams

Remarks

https://t.me/testseries4exams
9
Download All PDF form here :- https://t.me/pdf4exams

Remarks

https://t.me/testseries4exams
10
Download All PDF form here :- https://t.me/pdf4exams

Remarks

https://t.me/testseries4exams
11
Download All PDF form here :- https://t.me/pdf4exams

2. (a) History is a gradual progress of defining the identity of society. Elaborate it with
special reference to Indian history writing. (20 Marks)

(b) Through the Paleolithic painting draw a sketch of the life-ways of Paleolithic hunters
and gatherers. (15 Marks)

(c) Write a short note on the Social organisation and Settlement Pattern of Megalithic
culture in India. (15 Marks)

Remarks

https://t.me/testseries4exams
12
Download All PDF form here :- https://t.me/pdf4exams

Remarks

https://t.me/testseries4exams
13
Download All PDF form here :- https://t.me/pdf4exams

Remarks

https://t.me/testseries4exams
14
Download All PDF form here :- https://t.me/pdf4exams

Remarks

https://t.me/testseries4exams
15
Download All PDF form here :- https://t.me/pdf4exams

Remarks

https://t.me/testseries4exams
16
Download All PDF form here :- https://t.me/pdf4exams

Remarks

https://t.me/testseries4exams
17
Download All PDF form here :- https://t.me/pdf4exams

Remarks

https://t.me/testseries4exams
18
Download All PDF form here :- https://t.me/pdf4exams

Remarks

https://t.me/testseries4exams
19
Download All PDF form here :- https://t.me/pdf4exams

3. (a) To what extent you think that the religious pattern of Indus Valley Civilization
continued even in the later periods? (20 Marks)

(b) In architecture there is no real trace of relationship between the brick houses of
Harappa and the temples of Historical age, whether the earliest sculptures of
historical times, on the other hand shows a great likeness to that of Harappa.
Elucidate. (15 Marks)

(c) "There is an integral connection between urbanism and state societies." Analyse the
statement with respect to urbanism in Indus Valley Civilisation. (15 Marks)

Remarks

https://t.me/testseries4exams
20
Download All PDF form here :- https://t.me/pdf4exams

Remarks

https://t.me/testseries4exams
21
Download All PDF form here :- https://t.me/pdf4exams

Remarks

https://t.me/testseries4exams
22
Download All PDF form here :- https://t.me/pdf4exams

Remarks

https://t.me/testseries4exams
23
Download All PDF form here :- https://t.me/pdf4exams

Remarks

https://t.me/testseries4exams
24
Download All PDF form here :- https://t.me/pdf4exams

Remarks

https://t.me/testseries4exams
25
Download All PDF form here :- https://t.me/pdf4exams

Remarks

https://t.me/testseries4exams
26
Download All PDF form here :- https://t.me/pdf4exams

Remarks

https://t.me/testseries4exams
27
Download All PDF form here :- https://t.me/pdf4exams

SECTION-B

4. (a) Upanishads marked a great shift in the philosophical thoughts in the Later Vedic
Period. Analyse. (20 Marks)

(b) Critically analyse the Panini's work on Sanskrit grammar. (15 Marks)

(c) "During the later Vedic Period some remarkable changes could be noticed in the
treatment of vis." Discuss. (15 Marks)

Remarks

https://t.me/testseries4exams
28
Download All PDF form here :- https://t.me/pdf4exams

Remarks

https://t.me/testseries4exams
29
Download All PDF form here :- https://t.me/pdf4exams

Remarks

https://t.me/testseries4exams
30
Download All PDF form here :- https://t.me/pdf4exams

Remarks

https://t.me/testseries4exams
31
Download All PDF form here :- https://t.me/pdf4exams

Remarks

https://t.me/testseries4exams
32
Download All PDF form here :- https://t.me/pdf4exams

Remarks

https://t.me/testseries4exams
33
Download All PDF form here :- https://t.me/pdf4exams

Remarks

https://t.me/testseries4exams
34
Download All PDF form here :- https://t.me/pdf4exams

Remarks

https://t.me/testseries4exams
35
Download All PDF form here :- https://t.me/pdf4exams

5. (a) How far do you agree that the prime concern of Buddhism was society than the
religion in Post Vedic era. (15 Marks)

(b) "The social stratification was the most important prerequisite for the establishment
of state system, which could be in Ancient India." Critically evaluate. (15 Marks)

(c) What were the different trends of the thinking which emerged in the wake of the
socio-religious ferment during Mahajanpada period? (20 Marks)

Remarks

https://t.me/testseries4exams
36
Download All PDF form here :- https://t.me/pdf4exams

Remarks

https://t.me/testseries4exams
37
Download All PDF form here :- https://t.me/pdf4exams

Remarks

https://t.me/testseries4exams
38
Download All PDF form here :- https://t.me/pdf4exams

Remarks

https://t.me/testseries4exams
39
Download All PDF form here :- https://t.me/pdf4exams

Remarks

https://t.me/testseries4exams
40
Download All PDF form here :- https://t.me/pdf4exams

Remarks

https://t.me/testseries4exams
41
Download All PDF form here :- https://t.me/pdf4exams

Remarks

https://t.me/testseries4exams
42
Download All PDF form here :- https://t.me/pdf4exams

Remarks

https://t.me/testseries4exams
43
Download All PDF form here :- https://t.me/pdf4exams

6. (a) "Kautilya's thought which we come to know through Arthsashta, does not have a
pragmatic approach." In the light of given statement critically evaluate Arthsashta.
(15 Marks)

(b) Chaityas and viharas played major role for the integration and consolidation of
Buddhism during Mauryan period. Explain. (15 Marks)

(c) Evaluate the knowledge on medical science of Ancient Indian People.


(20 Marks)

Remarks

https://t.me/testseries4exams
44
Download All PDF form here :- https://t.me/pdf4exams

Remarks

https://t.me/testseries4exams
45
Download All PDF form here :- https://t.me/pdf4exams

Remarks

https://t.me/testseries4exams
46
Download All PDF form here :- https://t.me/pdf4exams

Remarks

https://t.me/testseries4exams
47
Download All PDF form here :- https://t.me/pdf4exams

Remarks

https://t.me/testseries4exams
48
Download All PDF form here :- https://t.me/pdf4exams

Remarks

https://t.me/testseries4exams
49
Download All PDF form here :- https://t.me/pdf4exams

Remarks

https://t.me/testseries4exams
50
Download All PDF form here :- https://t.me/pdf4exams

Remarks

https://t.me/testseries4exams
51
Download All PDF form here :- https://t.me/pdf4exams

7. (a) What were the impacts of Persian invasion in India? Is the impact could be noticed
in Ashokan pillars as well? (15 Marks)

(b) What were the major nastika philosophies in Ancient India and in what ways it
differed from astika philosophies. Write with special reference to Lokayat sect.
(15 Marks)

(c) "After the Kushanas, Mahayana Buddhism deviated a lot from the original Buddhism,
but even though it attracted the mass a lot." Critically analyse. (20 Marks)

Remarks

https://t.me/testseries4exams
52
Download All PDF form here :- https://t.me/pdf4exams

Remarks

https://t.me/testseries4exams
53
Download All PDF form here :- https://t.me/pdf4exams

Remarks

https://t.me/testseries4exams
54
Download All PDF form here :- https://t.me/pdf4exams

Remarks

https://t.me/testseries4exams
55
Download All PDF form here :- https://t.me/pdf4exams

Remarks

https://t.me/testseries4exams
56
Download All PDF form here :- https://t.me/pdf4exams

Remarks

https://t.me/testseries4exams
57
Download All PDF form here :- https://t.me/pdf4exams

Remarks

https://t.me/testseries4exams
58
Download All PDF form here :- https://t.me/pdf4exams

Remarks

https://t.me/testseries4exams
59
Download All PDF form here :- https://t.me/pdf4exams

8. (a) Discuss the changing social pattern in post Mauryan and Gupta period with special
reference to verna and jati. (20 Marks)

(b) "The Gupta period could be called a golden age for the development in astronomy
and mathematics." Examine. (15 Marks)

(c) Over the time it became difficult for the individual to upgrade in the social order
but social mobilization was always evident in Indian society. Critically Examine.
(15 Marks)

Remarks

https://t.me/testseries4exams
60
Download All PDF form here :- https://t.me/pdf4exams

Remarks

https://t.me/testseries4exams
61
Download All PDF form here :- https://t.me/pdf4exams

Remarks

https://t.me/testseries4exams
62
Download All PDF form here :- https://t.me/pdf4exams

Remarks

https://t.me/testseries4exams
63
Download All PDF form here :- https://t.me/pdf4exams

Remarks

https://t.me/testseries4exams
64
Download All PDF form here :- https://t.me/pdf4exams

Remarks

https://t.me/testseries4exams
65
Download All PDF form here :- https://t.me/pdf4exams

Remarks

https://t.me/testseries4exams
66
Download All PDF form here :- https://t.me/pdf4exams

Remarks

https://t.me/testseries4exams
Download All PDF form here :- https://t.me/pdf4exams

Scanned by CamScanner
https://t.me/testseries4exams
Download All PDF form here :- https://t.me/pdf4exams

History Test Series 2018

www.iasscore.in

HISTORY
Answer Hints: Test No. 2
1. Identify the following places marked on the map supplied to you and write a short note of
about 30 words on each of them in your Question-cum" Booklet. Locational hints for each
of the places marked on the map are given below seriatim.
1. A Jain site- Pavapuri
2. A Buddhist educational cente- Nalanda

E
3. A port site- Sopara
4. An Ancient capital- Kamrup
5. A Palaeolithic site - Hiran valley
OR
6. A historical site- Vidisha
7. A Harappan site- Kotdiji
SC

8. An ancient capital- Champa


9. A Manufacturing centre in Ancient India- Uraiur
10. A later Vedic site- Bhagwanpura
11. A Mesolithic site- Adamgarh
GS

12. A Chalcolithic site - Ahar


13. A Buddhist centre - Vikramshila
14. A trade centre in Ancient India-Suvarnagiri
15. An ancient capital - paithan
16. A port site- Bharuch
17. Rock-cave art centre- Bagh
18. An ancient capital- Patliputra
19. A political and cultural center- Madurai
20. An ancient town- Dwarka
2. (a) History is a gradual progress of defining the identity of society. Elaborate it with
special reference to Indian history writing.
History is a discipline which studies the past of the society specially those events which leaves some
impact on the society. It does the analysis of the continuity and change in the society. In this process
Hints: History [1]

https://t.me/testseries4exams
Download All PDF form here :- https://t.me/pdf4exams

it defines the identity of the society. But the identity has not been fixed as different school defines it
in their own way. In this process, their biased view also associated them so it is very hard to say
that any one of them has defined the real identity.
Indian history writing is having the same problem. In 18th Century the colonial people when got
the nizamat (administrative) right in Bengal, they had to understand the Indian society. In order to
understand the society they established many organisations such as Asiatic Society of Bengal, Asiatic
society of Great Britain e.t.c. which translated many Indian classical books. On the basis of these
translations they started writing history of India such as manusmriti, Bhagvat Gita and some other
religious texts. But they were worried of the fact that British civilians coming to India were getting
Brahmanised and developing inferiority complex.
To overcome this problem and to inculcate a sense of superiority complex among the British officers
about western culture, they started writing a distorted version of Indian history. They desperately
criticized the Indian society and showed it uncivilized in order to justify their rule on India. They
came up with such conclusions like Indians are never capable of ruling by their own that is why

E
earlier Aryans than Muslims came from outside and ruled on India and British are here for the
same. Most of the missionary writings were more driven by the desire to preach their faith rather
than provide objective narration of history- like Al-Beruni did. Christian Missionaries and European
OR
Historians were more interested in learning and writing about Indian history in order to depict its
flaws and prepare the ground for evangelical activity.
Hence some European historians launched a mission to discredit Indian History and it was very
obvious that it hurt the Indian sentiments and the educated Indian intelligentsia of the nineteenth
century was horrified at the distortions of the ancient Indian history by these European Historians.
SC

They decided to reconstruct Ancient Indian history in such manner- to make a case for social
reform, self-governance and Hindu revivalism.
In the late nineteenth century some scholars like Rajendra Lai Mitra, R.G. Bhandarkar, and V.K.
Rajwade tried to look at the ancient Indian history from the Indian point of view.
The contributions of all these great scholars helped in clearing the mist built by the missionaries and
the imperialist historians. They came up with a different version of history which was actually an
GS

effort to answer the colonial criticism instead of writing an objective history of India.
The Marxist school of historiography used to be the most influential school of history in the second
half of the last century. Despite the inherent contradiction and total failure of Marxist model of
history writings it is academically important to discuss it and give respect to the contributions it has
made. The Marxists believe in universal laws and stages of history. People like D D Koshambi, R S
Sharma have used the Marxist vision of historiography to write down the history of India.
Some others schools also came up in the second half of the Nineteenth century as Subaltern,
Cambridge e.t.c. the Subalterns are basically trying to write down the history of those sections
which have been neglected in main stream of history writing. So it can be seen that people have
always created with the biased vision and in that process defined the identity of the society in their
own way. This process is yet not over and still scholars are defining the society in their own way.
2. (b) Through the Paleolithic painting draw a sketch of the life-ways of Paleolithic hunters
and gatherers.
The life-ways of Paleolithic people living in different parts of the subcontinent were based on their
adaptation to their specific environments. However there were some basic similarities in the lives of
all the hunting gathering communities. These days' ethnographical studies are being done to study

[ 2 ] Hints: History

https://t.me/testseries4exams
Download All PDF form here :- https://t.me/pdf4exams

their life-ways. But apart from the ethnographical studies the paintings of that phase are also giving
a fair understanding of the lives of those people. Although these understandings are based upon
the assumptions as no other sources are there to approve it.
• In India, remnants of rock paintings have been found on the walls of caves situated in
several districts of Madhya Pradesh, Uttar Pradesh, Andhra Pradesh, Telangana, Karnataka,
Bihar and Uttarakhand.
• Some of the examples of sites early rock paintings are Lakhudiyar in Uttarakhand, Kupgallu
in Telangana, Piklihal and Tekkalkotta in Karnataka, Bhimbetka and Jogimara in Madhya
Pradesh etc.
• Paintings found here can be divided into three categories: Man, Animal and Geometric
symbols.
Life-ways of Paleolithic people
• Shelters made of rock, branches, grass, leaves or reeds

E
• Shelters nearby the waterbody
• Butchery sites and factory sites OR
• Band or small communities
• Livelihood- hunting and gathering
• Animals with them
SC

• Religious beliefs
• Artistic and social life
2. (c) Write a short note on the Social organisation and Settlement Pattern of Megalithic
culture in India.
It is not archaeology but anthropology, which provides us evidence to assume the possibility of
production relations transcending clan ties and kinship in such remote periods of tribal descent
GS

groups. By and large they point to the material culture of diverse forms of subsistence such as
hunting/gathering and shifting cultivation besides the production of a few craft-goods,
Though there was commonality in the idea of megalithism and the associated assemblages, the
variations observed in the external and internal features of the burials reflect that the Iron Age
society of the megalithic people was not a homogenous entity. Some of the relatively huge burial
types are suggestive of status differentiation and ranking of the buried individuals as discussed
earlier. Differences in the types and contents of the burials suggest that there was some sort of
disparity in the attributes of the buried individuals. The number of more elaborate burials like the
multi-chambered rock-cut tombs at many sites, are limited. Moreover, these have yielded rare artefacts
made of bronze or gold. On the other hand, many of the burials are simple urn burials with a very
few artefacts. The variety, high quality generally assume that "an individual treatment at death
bears some predictable relationship to the individual's state in life and to the organisation of the
society to which the individual belonged".
The megalithic people lived in villages consisting of a sizeable population. Though they had a bias
for the urban life, they were slow in building huge cities like their contemporaries in the Gangetic
Valley. The size of the population is indicated by the organised mass of manual labour that was
available for transporting and housing massive blocks of stone in the construction of cists, dolmens
and other types of megaliths, or in erecting large rubble and earthen mounds across the water

Hints: History [3]

https://t.me/testseries4exams
Download All PDF form here :- https://t.me/pdf4exams

courses for storing up rain waters for irrigational purposes. The large size of population is further
attested by the fact that extensive burial grounds with numerous graves, many of them containing
the remains of more than one individual, and occasionally of as many as 20 or more individuals,
have been found.
The houses in which the megalithic people lived probably consisted of huts with thatched or reed
roofs, supported on wooden posts as indicated by the presence of postholes in the excavated sites.
At Brahmagiri and Maski were found postholes indicating the presence of timber construction for
ordinary buildings. Some scholars suggest an advance stage of wooden architecture during the
megalithic period.
An increase in the size and number of settlements during megalithic period from the preceding
neolithic/chalcolithic phase and growing use of different metallic resources was certainly not an
independent development. This can be perceptively observed, as it has been argued in the effect of
the spread of plough cultivation which produced major alterations in the structure and distribution
of settlements. Although it is difficult to substantiate this point further in the absence of studies
concerning land-use patterns during the megalithic period. However, an analysis of the available
data on the locational context and the distribution patterns of these sites strongly indicate a growing

E
inclination towards intensive-field method. He suggests village transhumance on the basis of the
location of most of the settlement sites either on the banks of major rivers or on their major tributaries
OR
and that of most of the burial sites within a distance of 10-20 km from major water resources. The
maximum concentration of sites in river valleys and basins and preference shown towards occupying
black soil, red sandy-loamy soil zones also supports this contention. The distribution pattern of
these sites in rainfall zones where the average annual precipitation is 600-1500 mm, also hints to
the same conclusion.
3. (a) To what extent you think that the religious pattern of Indus Valley Civilization continued
SC

even in the later periods?


NOTE- REFER THE DISCUSSION
3. (b) In architecture there is no real trace of relationship between the brick houses of Harappa
and the temples of Historical age, whether the earliest sculptures of historical times,
on the other hand shows a great likeness to that of Harappa. Elucidate.
GS

In architecture there is no real trace of relationship between the brick houses of Harappa and the
stone temples of Hindu India. The earliest sculpture of historical times, on the other hand,
shows a generic likeness to that of Harappa, which we have already described From the end of the
Indus cities to the rise of the Mauryas over, a millennium elapsed, with no surviving work of art to
fill it. Somewhere in North India the art of sculpture no doubt in perishable materials, was certainly
kept alive. The patronage of the Mauryan emperors the influx of western influence and growing
material prosperity led to its revival, and to the making of stone figures and reliefs which are preserved
to this day.
The capitals of Ashoka's columns, some of which were perhaps made before his reign, are the
earliest important sculptures after those of the Indus cities. They are not characteristic of Indian
sculpture, though they contain many native features. The famous lions of the Sarnath column and
the less famous but more beautiful bull of column of Rampurva (pl. XXIIIb) are the work of realistic
sculptors, owing something to Iranian and Hellenist tradition. Yet, if we did not know that the
possibility of Western influence existed, we might suggest that the animal sculptures of the columns
were those of school directly descended from the engravers of the Indus seals which also show a
realistic treatment very unusual for so early a civilization. The abaci of the capitals perhaps show
native influence more clearly than the crowning figures, and bear animals in lively postures, wheels,
representing both the Buddha and the Mauryan World-emperor, and floral and foliate designs in
which typical, Indian motifs appear side by side with some borrowed from the West. Other than

[ 4 ] Hints: History

https://t.me/testseries4exams
Download All PDF form here :- https://t.me/pdf4exams

the pillars there are few remains of the Mauryan school, with its high polish and fine finish. One
beautiful figure, the "Didarganj Yaksi", bears the distinctive brilliant polish of the school, but the
treatment suggests that it is post- Mauryan. The yaksi bears a cauri, or ceremonial yak's tail fly-
whisk with which kings and gods were fanned; this shows that she was made as the attendant on
another figure or a sacred object, which has now vanished.
3. (c) "There is an integral connection between urbanism and state societies." Analyse the
statement with respect to urbanism in Indus Valley Civilisation.
A city is a node where population chooses to concentrate, to create a settlement larger and more
dense than most other contemporary settlements, not in order to make food production more efficient,
but because of an engagement in non-subsistence activities such as crafts or trade, administration
or ritual services.
Let us take the example of a small rural settlement. According to geographers, an area of land (upto
a distance of 4 km) round a settlement would be used for locating agricultural fields. Areas further
away would be used for other purposes such as pasture lands, forests and for other requirements,
as it would be disadvantageous to locate fields more than an hour's walking distance away from

E
the settlement. This agricultural land would provide livelihood for the inhabitants of the village. If
land for fields were available only further away, one would find fissioning of the village, with the
'hiving-off' or 'budding-off' of daughter settlements. In a nucleated settlement (a city/town) on the
OR
other hand, that amount of agricultural land that can conveniently be cultivated is unlikely to
suffice for the entire population. Hence, there is a greater significance in the engagement of non-
subsistence activities.
Why is there a need for population to nucleate? There could be ritual or political reasons, in an area
being a ritual or political centre, or for defence purposes or to make non-agricultural production
SC

and transactions more efficient. In the case of the latter, let us contrast again a rural settlement from
an urban one. In a rural landscape, each settlement may have a single potter or some may have no
potter. Each potter would work in isolation, procuring his own raw materials, like clay, fuel and so
forth, building his own facilities, and distributing his own products in the village and to nearby
villages. In the case of nucleated settlement on the other hand, potters could cooperate to procure
raw materials, share facilities like the kiln and distribution networks. Moreover, the larger consuming
population would provide for expansion in production and perhaps enable a greater tendency for
GS

specialization.
Socially an urban situation provides for residence within the settlement as being more significant
than kinship relations. Due to nucleation of population, dealings within the city are more likely to
be between strangers, unlike in a rural settlement. This would make way for changes in social
structures and for the existence of a state.
Mohenjodaro and Harappa are clearly the two largest Harappan settlements. We will take the case
of Mohenjodaro to illustrate a Bronze Age city. Mohenjodaro is divided into two sections, a higher
and a smaller western section termed the Citadel and a lower and larger area to its east called the
Lower Town. These two sections are clearly spatially separated and may have been individually
walled. Most large seemingly special purpose (non-residential) structures locate on the Citadel (such
as the storage facility, the Great Bath), though some large structures occur in the Lower Town too.
This may indicate the special nature of the Citadel as well as a possible attempt to segregate its
functions and inhabitants from the rest of the city.
Social heterogeneity can be inferred from the lack of uniformity in house types and sizes. Within the
Lower Town, large and small houses occur together. Material culture too indicates differences in
consumption. For example, a long bead in carnelian (a favoured red coloured stone) appears to
have been rare and copied in clay. Mohenjodaro also gives evidence for metallurgy, shell working,
bead making and the production of seals and weights. These crafts are not practiced in every
household, and many among them clearly required complex skills and facilities.
Hints: History [5]

https://t.me/testseries4exams
Download All PDF form here :- https://t.me/pdf4exams

By and large scholars concur that there is an integral connection between urbanism and state
societies. A state need not have urban centres, but the opposite does not generally hold true while
the existence of urban centres generally necessitates a state structure, though here again there can
be exceptions.
4. (a) Upanishads marked a great shift in the philosophical thoughts in the Later Vedic
Period. Analyse.
From early Vedic period to later Vedic period may changes could be noticed. These changes were in
the field of their geographical settlement, polity, economy, society and religion as well. As far as the
changes in the religious pattern are concerned, it was a journey from Rigveda to Upnishadas. The
Vedas were composed to appease the god just for the sake of material benefits. Most of the hymens
were dedicated to god just too demand some material things as cows, son, food grains etc. but in
upnishadas different short of concerns could be noticed which could be termed as metaphysical
like who is god, in what way the universe has been created, what is the ultimate reality, what is
salvation. There are many interpretations for this as many complications, such as rituals, sacrifices
increased in high manner in Vedic scriptures and people were not capable of adjusting with this

E
and so they were looking for an alternative within Brahminism. In a different argument it has been
said that in the changed scenario where society has gained much development in polity and economy,
some people could not adjust with this and they moved towards the spiritual life. But whatever the
OR
reason was, this metaphysical concern can be seen at the end of the Later Vedic Phase
4. (b) Critically analyse the Panini's work on Sanskrit grammar.
The great grammar of Panini effectively stablised the Sanskrit language. Though its fame is much
restricted by its specialized nature, there is no doubt that Panini's grammar is one of the greatest
intellectual achievements of any ancient civilization, and the most detailed and scientific grammer
SC

composed before the 19th Century in any part of the world. The work consists of over 4000
grammatical rules, in a sort of shorthand, which employs single letters or syllables for the names of
the cases, moods, persons, tenses, etc. in which linguistic phenomena are classified. The great terseness
of Panini's system makes his work very difficult to follow without preliminary study and a suitable
commentary. Later Indian grammars are mostly commentaries on Panini, the chief being the
Mahabhasya of Patanjali and Kasika Vritti of Jayaditya and Vamana.
Some later grammarians disagreed with Panini on minor points, but his grammar was so widely
GS

accepted that no writer or speaker of Sanskrit in courtly or brahmanic circle dared seriously infringe
it. With Panini the language was fixed, and could only develop within the framework of his rules.
It was from the time of Panini onwards that the language began to be called Samskrta.
Paninian Sanskrit, though simpler than Vedic, is still a very complicated language. Every beginner
finds great difficulties in surmounting Panini's rules of euphonic combination, the elaboration of
tendencies present in the language even in the Vedic times. Every word of a sentence is affected by
its neighbors. There are many rules of this kind which complicate the language. As long as it is
spoken and written a language tends to develop, and its development is greatly in direction of
simplicity. Owing to the authority of Panini, Sanskrit could not develop freely in this way. Some of
the minor rules, such as those relating to the use of tenses indicating past time, were quietly ignored,
and writers took to using imperfect, perfect and aorist indiscriminately; but Panni's rules of inflexion
had to be maintained. The only way in which Sanskrit develop away from inflexion was by building
up compound nouns to take the place of the clauses of the sentences.
4. (c) "During the later Vedic Period some remarkable changes could be noticed in the
treatment of vis." Discuss.
The Later Vedic period is known for transformation and changes with respect to the previous
period. The changes could be noticed in various aspect such as political, social.

[ 6 ] Hints: History

https://t.me/testseries4exams
Download All PDF form here :- https://t.me/pdf4exams

Perhaps the most striking change we notice is in the treatment of the vis. The term was still used to
refer to the entire community. For instance, on major ritual occasions such as the rajasuya and the
asvamedha, the sacrificer who was consecrated through the abhiseka was often proclaimed as the
raja of the vis. At the same time, the vis was often used to designate a residual category, distinguishing
it from the brahmana or the priest on the one hand, and ksatriya/rajanya (the ruler and/or his
supporters) on the other. (The term vaisya, meaning of or belonging to the vis, owes its origin to this
root).
This residual category was now, moreover, often identified as one that could be legitimately exploited.
The most vivid imagery of this occurs in the notion of the raja/ksatriya as visamatta, literally, the
eater of the vis, suggesting that the raja could appropriate the resources generated by the vis at will.
In the earlier situation, the vis seems to have had some claims to a share of the booty obtained from
raids. Now, this relationship was replaced by one of appropriation. At the same time, the raja was
still not in a position to claim a regular tax, but depended on less structured mechanisms for garnering
resources.
Rituals were also used as occasions to both construct, and represent, and perhaps, by extension,

E
ensure what was regarded as the ideal relationship between the ksatra and the vis. From the point
of view of the raja/ksatriya, the vis was ideally to be supportive but subordinate. So we find the use
of rituals and mantras to ensure this. However, there are indications that such ritual manipulations
OR
may not have always worked. One of the constant fears expressed in the texts is that the vis might
leave the ksatriya. Can we suggest that this might actually refer to situations where people might
migrate from the realm of a raja that have become excessively extortionate, thus depriving him of
resources, labour, as well as armed support?
5. (a) How far do you agree that the prime concern of Buddhism was society than the religion
SC

in Post Vedic era


Normally it has been perceived that Buddhism was religious philosophy Buddhism had many such
elements which proves that Buddhism was a more social philosophy.
In 6th Century BC when Buddhism emerged when many changes were taking place specifically in
economy. In the lower Gangetic plain much surplus has been produced and life became more
materialistic as compared to the previous eras. This materialism increased the miseries (dukkha) of
GS

the human being and miseries of human being been the central idea of Buddhist philosophy which
is known as Chatur Arya Satyani.
Apart from this many social issues were picked by Buddhism like slavery, women liberation, money
landing system etc. they also discarded the disparities among the society and Buddhism entitled
everyone for nirvana.
5. (b) "The social stratification was the most important prerequisite for the establishment
of state system, which could be in Ancient India." Critically evaluate.
As one enters the age of the Buddha many of these limitations were overcome. The introduction of
iron in agriculture helped deeper ploughing and the breaking of the hard soil in the mid-Ganga
plains. Iron was also used in various crafts and the making of metallic money, i.e., the punch
Marked coins. Almost simultaneously wet paddy transplantation came to be practiced in this
naturally rice growing area. Cumulatively these developments led to surplus produce, which in
turn sustained trade, taxes and the emerging stratified society, with its administrative functionaries,
ideologues and wage labourers. Dharmasutra literature justified varna divisions and institutionalised
inequality. Vaisyas and sudras bore the brunt of carrying out production and provided the necessary
revenue and labour to uphold the king's men, army personnel, priests, ideologues and so on.
Buddhism too recognised and endorsed many of these developments. There are references to minsters
and armies in the context of Magadha and Kosala. The presence of officials such as balisadhaka
Hints: History [7]

https://t.me/testseries4exams
Download All PDF form here :- https://t.me/pdf4exams

and karakara, for example, suggests that taxes like bali and kara were collected. Thus, by the sixth-
fifth centuries B.C. territorial states emerged in northern India.
The above mentioned perspective had been criticized largely on two counts. First, it is said that the
final emergence of states has been explained with reference to some kind of technical determinism
in what appears to be an iron-productivity surplus-state formation line of argument. Secondly, the
emergence of the varnas and their assigned roles, either as receivers of taxes and gifts or providers
of produce and labour, has not been fully explained.
That brings us to Romilla Thapar's explanation of the emergence of states. She refers to anthropological
concepts like lineage society and house-holding economy to explain the evolution of the hierarchically
structured varna society, and her emphasis is on the interplay of multiple processes of change,
bearing on state formation. It is said that Vedic literature is replete with references to lineage terms,
viz., gotra, vraja, etc. Lineage groups comprised members of the senior (rajanya) and junior (vis)
lineage. The senior lineage both controlled and had greater access to community resources, though
in principle there was collective ownership of land by the lineage group. In course of time by
characterising the seniority based on genealogical superiority as one premised on the ideology of

E
patrilineal descent the rajanya asserted its authority. It emphasised endogamy to claim purity, and
flowing from it asserted its exclusivity and superiority. The differentiation between members of the
OR
senior and junior lineage increased with the transition to the later vedic period.
The emergence of a socio-economic form approximating what is known as householding economy
is seen to have hastened the process of internal differentiation and the dissolution of lineage
organisation during the later vedic times. The household comprised three to four generations of
family members who may have resided in one or more than one house, but for purposes of production,
consumption and rituals formed one single unit. The extended family gradually began to exercise
SC

right on the land it cultivated, theoretically though such land was initially allotted to the community
for its use in cultivation. In situations where the extended family labour was not sufficient to work
the land, the non-kin members who were not related to the family by Kinship ties were roped in for
agricultural activities. These people need not be confused with wage labour. They were practically
a part of the family, participated in all family activities except the family rituals. In the long-term as
land allotted for cultivation was transformed into private property such retainers, who were some
sort of family inheritance and may have emerged out of defeated and dispossessed peoples, were
GS

reduced to family servants. The rajanya/kshatriya and vaisya evolved from the senior and junior
lineages respectively. Those relegated to the position of labourers and artisans become Sudras. Because
the extended families within the given socio-economic structure generally incorporated three-four
generations it allowed younger generations to move out, clear and settle in new lands in conditions
of population pressure. There are literary references to the fissioning off among communities as a
consequence of such developments. Such tendencies facilitated the process of agrarian expansion
and extended the frontier of peasant activity. Thus, within the framework of the house-holding
economy one comes to understand the transition from lineage society to a complex society and the
state.
In the final stages leading to the emergence of the state Thapar, eschewing simple mechanical
explanations, focuses on the mutually interactive nature of the processes. Environment, technology,
social stratification, surplus, urbanisation and ideology, among others, were important factors in
the making of the state, but it is difficult to prioritize them or identify the single most important
factor. Surplus, for example, was related to social and political hierarchies and the need of the non-
producers to live off the produce of others. Similarly, it was linked to the distribution of the produce.
In brief, society does not produce a surplus simply because of the availability of a given technology.
It is the result of a combination of factors. The relationship between social differentiation, urbanisation
and ideology too are quite complex. Powerful contemporary religious ideas and systems (Buddhist)

[ 8 ] Hints: History

https://t.me/testseries4exams
Download All PDF form here :- https://t.me/pdf4exams

played an important role in shaping the nature of the emerging state systems - gana sanghas and
monarchies. The Buddhist Sangha (monastic institution) characterised by its egalitarian ideas was
useful to the early states because it was able to integrate the varied groups across caste and clan
lines. The Sangha too depended for its sustenance on the existence of a strong state. Kings like
Ajatsatru of Magadha and Ashoka Maurya extended patronage to Buddhism. In this analysis it is
also argued that the mahajanapadas were either gana-sanghas or monarchies. While in the so
called republics of Northeastern India (Malla, Vajji) the process of transition to powerful centralised
state was slow owing to the common ownership of land by the kshatriya clans (which blocked the
possibility of land revenue appropriation) whereas the territorial states in the upper Ganga plains
(Kurus) could not easily shake off the later Vedic legacy of rituals, cattle sacrifice and wasteful
consumption (which hindered the rise of strong states), those like Kosala and Magadha which
were located in the midGanga plains were characterised by no such limitations. In addition, Magadha
had the advantage of rich soil, gentle gradient towards Ganga, a history of rice cultivation, good
rainfall, irrigated land, Bandhs used as water reserves, several rivers like the Son, Gandak, etc.,
which could also be used for communication and trade, and it was close to the mines and minerals
of Dhalbhum and Singhbhum. The forest of Rajmahal hills were used for procuring timber and

E
were also the habitat of elephants. Magadha controlled the Dakshinapath (trade route) and all
routes on the southern bank of the Ganga were linked to Magadha. The states that emerged in this
OR
part of northern India were evidently more viable and strong. They could sustain greater populations
and generate the necessary taxes to meet the requirements of the state.
5. (c) What were the different trends of the thinking which emerged in the wake of the socio-
religious ferment during Mahajanpada period?
As far as the religious context is concerned, Vedic Brahmanism was most prominent. However, old
SC

vedic religion had been reduced to an extremely formalized ritualism in the hands of Brahmans.
The emphasis was on the rigid observance of the rules prescribed for the performances of the sacrificial
rites, which had become the most important aspect of the religion. These sacrifices had become very
lengthy and expensive affair, affordable only to the high and rich classes of the society. The
Brahmanas, who monopolized the reading and interpretation of Vedas, were the most powerful
and prominent caste.
The changing politico-economic-social scenario naturally invoked much change at intellectual level.
GS

The period was marked by proliferation of ascetic sects with a wide range of ideas spanning from
annihilations (ucchedvada) to eternalism (sasvatavada) and from fatalism to the materialism. Though
the ascetic tradition and the ideas propagated by various sects had a long history, their appearance
in a concrete shape of definite sects in the 6th century B.C. was provoked by the changes of
contemporary society.
The emergence of imperial state against the decline of republics provoked much discussion. The
kingdoms came to be favoured by mainstream brahmanical society, which advocated the ideal of
'Universal Ruler'. However, another thought process protested against such domination, which
later on came to be manifested in the philosophy of Buddhism and Jainism. Some scholars even go
to the extent of suggesting that the political troubles of the age provided its more sensitive souls
with incentive to withdraw from the world, which accelerated the popularity of ascetic tradition.
The newly emerging castes of kshatriyas and vaishyas resisted the highest status claimed by the
brahmanas as they also aspired to rise in the social hierarchy. This conflict between the established
orthodoxy and the aspirations of new groups in the urban centers intensified the intellectual process,
which resulted in a remarkable richness and vigour in thought, rarely to be surpassed in the centuries
to come. Moreover, so many changes produced a sense of social stress and awakened the spirit of
questioning. The experience of social change and suffering is undoubtedly connected with the quest
of new pathways in religion and philosophy. Social change is an effect rather than a 'cause' of
spiritual change.
Hints: History [9]

https://t.me/testseries4exams
Download All PDF form here :- https://t.me/pdf4exams

There is no doubt that the older Vedic gods and sacrifices were conceived in the midst of rural and
agricultural landscape. In the new atmosphere of town-life, much of the symbolism of the older
religion derived from natural phenomena and pastoralagricultural functions would become dim,
the gods less convincing and the rituals obscure. The changing milieu witnessed the appearance of
new concepts and ideas.
Brahmanism: A sharp contrast had developed within brahmanism between formalistic, ritualistic
tendencies of Vedas and the new trend towards an esoteric and ascetic direction visible in the
Upanishadas. In these texts, the doctrine of ritual act was often replaced by that of knowledge and
sometimes by that of theistic devotion as well as moral conduct. Ritualism was receding, while
ascetic renunciation and creed of life of virtue and devotion was gaining importance. Thus, there
was growing cleavage of ideas within brahmanism itself.
Rise of Asceticism: A religious tradition parallel to brahmanism was the tradition of asceticism,
which was prevalent for a long time. The ultimate origins of this ascetic tradition are obscure. There
are traditions about ancient teachers, often in very remote period, but their historicity has not been
established as yet. Its definite history can be traced from 6th century B.C. The growth and spread of
asceticism in 6th century B.C. is the most characteristic feature of the new religious life that sprang

E
up. This new movement was led by the non-Brahmanas. Some Brahmanas also joined it, but they
thereby left the brahmanical tradition. The philosophers of these new schools of asceticism were
OR
called 'Sramanas' or 'Parivajrakas'. They were the men who had left the society and become
wanderers. They lived on alms and practiced rigorous penance of various forms. They rejected the
Vedas and the authority of the Brahmanas. They ridiculed the complicated rituals and tried to
show the absurdity of the Veda as a canon of ultimate truths by pointing out contradictions in it.
They declared that the entire brahmanical system was a conspiracy against the people by the
Brahmanas for the purpose of enriching themselves by charging exorbitant fees for rituals. In place
of this authoritarian tradition, the Sramanas sought to find explanations by own investigations.
SC

Even if the life of wandering in the forests was old, most of the philosophies of the period were new,
taking account of major changes at all levels of life. The establishment of organised communities of
Sramanas as opposed to individual wanderers was an innovation of the period. Debate, discussion
and teaching were important aspects of these schools. Audiences gathered around the new
philosophies in the kutuhala-salas, the place for creating curiosity.
Sramana Philosophy: Though there were a number of ascetic schools with independent concepts,
GS

most followed a general pattern. Their conception of the Universe was that it was a natural
phenomenon, evolving itself according to ascertainable natural laws. It was not subject to the control
of gods or a God and had not been created by such supernatural powers. If there were gods, as
some of them admitted might be the case, they were natural beings on a level with humans and
animals, inhabiting in different region, but just as subject to natural laws as humans. The gods were
not immortal, but lived and died as humans did. However, the most schools denied the existence of
God.
Most of the Sramanas believed in transmigration in some form, either of a 'soul' or of a stream of
consciousness from a dying body to a newly conceived one. By this period, Brahmanism also had
accepted this idea. Most of these schools regarded life as on the whole unhappy, filled with sufferings,
concluding that their aim should be, not to be reborn in it in better circumstances, which any way
would be temporary, but not to be reborn at all. Though the methods to achieve this aim differed,
the emphasis was primarily laid on the moral conduct and personal efforts of an individual, rather
than complicated rituals with the help of Brahmanas.
A number of such schools are mentioned in the literature of subsequent period. In Pali literature of
Buddhists, there is reference to 62 doctrinal views before Buddha, while the Jaina canons refer to
363 sects. However, of these, a few groups were most prominent and influential.
Ajivikas: This sect was founded by a group of prominent teachers in Kosala. The leader of this
school was Makkhali Gosala. Other important teachers were Purna and Kakuda. The Ajivikas
[10] Hints: History

https://t.me/testseries4exams
Download All PDF form here :- https://t.me/pdf4exams

believed in transmigration on a grand scale. Their key doctrine was that 'niyati' or impersonal
'destiny' governed all; such that humans had no ability to affect their future lives by their karma as
actions were not freely done, but were predetermined. The destiny controlled even the most
insignificant action of each human being and nothing could change this. Thus, they believed in
rebirth, but not in karma. They practiced rigorous asceticism such as fasting and nakedness.
Lokayatas: The followers of this school were materialists. The main spokesman was Ajita
Kesakambala. They denied any kind of self other than the one, which could be directly perceived.
Each act was seen as a spontaneous event without karmic effects and spiritual progression was not
seen as possible. Man was made of dust and returned to dust. Thus they denied soul, transmigration
and also destiny. This school was also known as Do-as-you-like school (yadrcchavada). They believed
that the aim of living beings was happiness and highest happiness was of pleasures of the senses.
Unlike other schools, they maintained that there was more happiness than unhappiness in life.
Later on, Charvaka became the prominent leader of this theory.
Skeptics: Their spokesman was Sanjaya Belatthaputta. They avoided commitment to any point of
view. They held that no conclusive knowledge was possible and did not even commit them to
saying that other people's views were wrong. One of the primary concerns of these Sramanas was

E
whether moral actions would have any affect on the person who performed them, in other words,
the existence and functioning of karmic cause and effect. If moral actions did have effects, then the
religious practitioners had to investigate how he might break his karmic bonds and free his mind or
OR
soul and achieve final release from the cycle of birth and rebirth. Such was the cultural milieu in
which Buddhism and Jainism rose.
Buddhism: Buddha (566-486 B.C.) was the Kshatriya prince of the republican clan of Sakyas and
was known as Siddhartha in his worldly life. He was born at Lumbini, on the Nepalese side of Indo-
Nepal border. After living a life of an aristocrat, he encountered sickness, suffering and death as
well as asceticism for the first time in his life through famous four visions of a sick, an old and a
SC

dead person and an ascetic. Highly dissatisfied with the transitory nature of life, he finally left his
house, wife and the child at the age of 29 and became an ascetic. He joined various ascetic groups
and followed different types of asceticism prevalent at the time. He wandered around for six years.
When nothing worked, he decided to discover his path through meditation. He achieved
enlightenment at the age of 35, while meditating under a tree at Bodhagaya. He gave his first
discourse at Sarnath, near Varanasi, where he gathered his first five disciples. For 45 years, he
wandered around, mainly in Bihar region, preaching his creed in the local language, Pali. The
GS

religion was soon adopted many important dignitaries of the period as well as a number of common
people. He died at the age of 80 years at Kapilavatsu after establishing his sect on firm footing.
Buddha promulgated a doctrine, which had all the main characteristics of the Sramana movement.
He rejected all authority except experience. One should experiment for himself and see whether the
teaching is true. The Universe is uncreated and functions on natural laws. It is in continuous flux.
He denied the existence of soul, though accepted the process of transmigration and karma. According
to him, in transmigration, the new life arises as part of the chain of events, which included the old.
The only stable entity was Nirvana, the state of infinite bliss. The aim of human life was to achieve
this nirvana and end transmigration. The path to achieve this aim constituted most important part
of teaching. The basic principles of Buddhism are Four Noble Truths: 1) world is full of suffering, 2)
suffering is caused by human desires, 3) renunciation of desire is the path to salvation, 4) salvation
is possible through Eightfold path, which comprised of eight principles, emphasizing on moral and
ethical conduct of an individual. Buddha preached the 'Middle Path', a compromise between self-
indulgence and self-defeating austerities.
The religion was essentially a congregational one. Monastic orders were introduced, where people
from all walks of life were accepted. Though Buddha was initially against the entry of women into
asceticism, an order of nuns was established eventually. Monks wandered from place to place,
preaching and seeking alms, which gave the religion a missionary flavour. The organisation of
Sangha was based on the principles of a gana-sangha.

Hints: History [11]

https://t.me/testseries4exams
Download All PDF form here :- https://t.me/pdf4exams

Jainism: Jainism has longer history than Buddhism. Jaina ideas are said to have been prevalent
since time immemorial as twenty-three tirthankaras or makers of fords are recorded to have lived
before Mahavira in remote past. Though the historicity of these tirthankaras is not proved, the 23rd
tirthankara, Parsvanatha could have been a historical personage of 8th century B.C. However, it
was Mahavira who reorganized the sect and provided it with historical basis. The sect was initially
known as 'Nirgrantha' ('knotless' or free from bonds), but later on came to be known as 'Jaina', after
Jina-the Conqueror, which refers to Mahavira.
The life of Mahavira (540-468 B.C.) has striking similarities with that of Buddha. He was also a
Kshatriya prince of Jnantrika clan, which was a part of famous Vrijji confederacy. He was born at
Kundugram, a suburb of Vaishali and was known as Vardhamana. In Buddhist texts, he is also
called Nataputra and Videhan, son of Jnatras and resident of Videha. He too, after living a life of an
aristocrat, renounced the world at a young age of 30. He practiced rigorous asceticism for twelve
years in search of truth. He wandered in Bihar and parts of Bengal. He finally achieved enlightenment
outside the town of Jambhiyagama after which he preached his doctrine for 30 years. He mainly
traveled in Bihar, spending maximum time at Vaishali and Rajagriha. He met with great success in
Bihar and parts of western Bengal also came under his influence. Many important personalities of

E
his time and rich merchants are said to have accepted his creed. Many ordinary people were also
brought into the fold. He found the orders of monks and nuns. He too preached in the local language,
Ardhamagadhi. He died at the ripe old age of 72 at Pawa.
OR
The Jainas also rejected the existence of God. According to the Jaina philosophy, the Universe is
uncreated and moves in a cyclic motion of decline and progress. During each epoch, twenty-four
tirthankaras are born who revive the Jaina religion. The universe functions through the interaction
of living souls (jivas) and five categories of non-living entities (ajiva), which are akasa, dharma,
adharma, kala and pudgala. Not only the human, animal, and vegetable organisms, but also things
like earth; fire and water have souls. By nature, the soul is bright, pure and conscious, but it gets
SC

covered by the matter of karma, which accumulates by any and every activity. Only by removing
this karma, one can achieve moksha or liberation from the cycle of transmigration, which is a state
of inactive bliss. The annihilation of karma comes through prevention of the influx and fixation of
karma in soul by careful, disciplined conduct of right knowledge, right vision and right conduct.
Unlike Buddhism, Jainism laid great emphasis on self-mortification and rigorous austerities, mainly
fasting. It differed from Buddhism and also Brahmanism in believing that full salvation was not
possible for the laymen as total abandonment was necessary for attaining nirvana. The path to
GS

nirvana was observance of five vows, non-killing (ahimsa), non-stealing (achorya), non-lying
(astyeya), non-possession (aparigrahara) and celibacy (brahmacharya). While Parsvanatha preached
the first four vows, Mahavira added the last one. The Jainas laid great emphasis on ahimsa and
formulated a number of rules for observing ahimsa in daily life.
Thus, the emergence of these two similar ascetic sects, which emphasized the transitory and painful
nature of human life and preached the salvation as the final solution, to be achieved by observing
moral conduct, entirely through an individualistic effort as against by complex rituals through a
priest, was a reaction to a changing society and an attempt to fulfill the needs of new society.
6. (a) "Kautilya's thought which we come to know through Arthsashta, does not have a
pragmatic approach." In the light of given statement critically evaluate Arthsashta.
Before evaluating Kautilyan thought, we must not forget the context in which the Arthasastra was
written. Kautilya was trying to create, almost single-handedly, order out of chaos, peace out of
war, a public state out of a corrupt one. That is why his ideas were extremely complex.
On the one hand he had suggested the use of all evil means; on the other hand he was obsessed by
the idea of creating a neat administrative system, town planning, problems of mines, rural and
urban colonialisation and settlement, with the creation of dams and canals no less than with a
monetary system and control of weights and measures. Obviously, such a man could not be preaching

[12] Hints: History

https://t.me/testseries4exams
Download All PDF form here :- https://t.me/pdf4exams

political gangsterism; he is not preaching that the end justifies the means, the only end, which
justifies the means, is the preservation and development off the state, punishment of the wicked
and protection of the good.
His great insight lies in the discovery that any agency entrusted with the task of maintenance of
order, acquisition of what has not been achieved so far, and distribution of surplus to the deserving
in society, required creation of an agency or authority which cannot be a common standard to
evaluate all human action, namely, the imperative of life in society, to continue without hurting
anyone in the process. Consequently, it means that nonviolence is good because as Bhishma said in
Mahabharata, anyone who is non-violent gives life breath to the universe.
Arthashastra, the great Indian classic on public administration, deals with the hierarchy of officials,
the merits and failings of bureaucracy including corruption, geographical divisions of the empire,
field administration through an all-purpose coordinating district overlord, land revenue and taxation.
The Arthashastra is comprised of 413 maxims. Of the fifteen chapters of the book, public
administration claims four.
Foreign relations and defence are discussed in nearly half of Arthashastra, which has little concern

E
for public administration. In brief Kautilya's work is a comprehensive handbook of imperial
administration and diplomacy. Also, it is remarkably practical and utilitarian and it advocates 'real
politics'. OR
Kautilya's Arthashastra is unashamedly practical. This work is the loudest proof of the practical
turn of the Indian mind. It is a manual of practical state craft for the day to day guidance of the
prince - that is the ruler. 'Arthashastra' means the science of politics and public administration.
What Kautilya calls 'Arthashastra', most other Indian writers of this shastra call by other name
namely 'nitishastra' or 'rajanitishastra'?
SC

The hallmark of the Arthashastra is the sharp acumen of Kautilya. The principles of public
administration implied in Arthashastra does not command as much attention as the machinery of
government does.
Kautilya, himself, does not give a definition of the science of public administration. In fact, the term,
'the science of public administration is of recent origin. According to the scholars of the ancient
society and polity, the executive - which is the administrative branch of the government - evolved
GS

much earlier than the judicial and the legislative branches.


According to Kautilya, the science of polity is the Arthashastra and it, mainly, treats public
administration. It is the combination of the science of wealth and the science of government.
The finance department and the other departments dealing with business and allied economic
activities form a vital part of the government.
All ancient Indian writers including Kautilya have invariably discussed the following topics such
as monarchy, constitution and functions of the ministry, principles of foreign policy, problems of
taxation, provincial and local administration, theories of the origin of the state and the elements of
state and miscellaneous theories.
Kautilya says that Crown property is the main basis of production and its profitable administration
is the chief preoccupation of the state. The concern of the state is all-embracing, replacing private
enterprise to an extent "never seen before or after in India".
Kautilya believed in a strong centralized state. For the good of the state, the king enjoyed full
freedom - meaning that he was free to practice treachery, deceit and sacrilege if necessary.
According to him, the constituent elements of the state are the following:
1. The king who exercised political authority (The Svamiri)

Hints: History [13]

https://t.me/testseries4exams
Download All PDF form here :- https://t.me/pdf4exams

2. The officials, who advised him on public administration (The Amatya)


3. Territory (Janapada)
4. The forts (The Durga)
5. The army (The Danda)
6. The treasury (The Kosh)
7. Allies (Mitras)
Among these, the king is the most critical element. He must attain superiority, pre-eminence and
over lordship in his kingdom. He is the personification of the state and its first citizen. He is the
embodiment of power. The principle of unity of command is reflected here.
In the eyes of Kautilya, the king and the state are indispensable - the latter is such an essential part
of society. Succession by heredity was the general rule though exceptions also did exist.
Kautilya says, an ideal king is one who possesses the highest qualities of leadership, intellect, energy

E
and other personal attributes. He must not be dilatory in his decision making, be stronger than his
counterparts of nearby countries and to be assisted by competent ministers.
OR
Arthashastra portrays a paternal king imbibed with a desire to promote the happiness and welfare
of his subjects. In the happiness of his subjects lays his happiness, in their welfare his welfare,
whatever pleases his subjects he shall consider as good.
Hence the king shall ever be active and discharge his duties, the root of wealth is activity and of evil
its reverse. The welfare of the people claims the first place in his eyes. The main function of the state
was to maintain law and order.
SC

According to Kautilya, the king was a servant of the state. A king has no personal likes; it is the likes
of the subjects (that should be followed by him). The king should surrender his individuality in the
interest of his duties and be virtually a constitutional slave.
The functions of the state were wide ranging and all-pervasive. This means that the public
administrative system designed in the state was necessarily large and sprawling. A large civil service,
to use the modern term for officials of that day, was needed to perform the myriad functions required.
GS

The following words of Kautilya speak of the concept of welfare state, which is embedded in the
directive principles of the constitution of India.
6. (b) Chaityas and viharas played major role for the integration and consolidation of Buddhism
during Mauryan period. Explain.
The use of the term Chaitya suggests a pre Buddhist sacred enclosure that was a regular part of the
worship in the gana-sanghas. The chaitya complex also at times contained a stupa, originally a
funeral monument and a memorial relic later on, which was the main object of worship in Buddhism
before the introduction of image worship. The chaitya cave consisted of either an apsidal, vaulted-
roofed or a square, flatroofed hall with the rock-cut stupa at one end having circumambulatory
formed by a row of pillars around and a verandah. A vihara was basically a hall with a number of
cells along all sides and with or without a verandah. These caves were simple with sparse decoration
in the form of ornamental pillars, elaborate façade and a few auspicious symbols occurring above
the cell doors. This visible form of architecture was possible due to the nature of the patronage
received by the sect from its patrons. These patrons were linked to the process by which the religion
spread across the parts of India in post Mauryan phase.
The spread of Buddhism to distant lands of peninsular India, central India and also to other countries
is often associated with the mechanism of expanding trade networks and empire building activities.
There is no doubt that it was primarily the proselytizing efforts of dynamic and enterprising monks,
[14] Hints: History

https://t.me/testseries4exams
Download All PDF form here :- https://t.me/pdf4exams

who ventured through unknown lands to preach the creed that led to the spread and popularity of
Buddhism in far-off lands. But the process of second urbanization, which spread from the Gangetic
valley to the rest of the country, with its growing trading networks, definitely accelerated the spread
of Buddhism.
The phenomenon of urbanization and trade, which started in 6th century B.C., gained momentum
in the subsequent centuries. The volume of trade increased immensely as the trade with the
Mediterranean world, which probably existed for a long time, was intensified. By 3rd-2nd century
B.C., almost all parts of the country experienced a phase of urbanism, accompanied by the emergence
of a powerful imperial state, agricultural expansion and growing economy characterized by
increased volume of trade, appearance of metal currency as well as craft specialization. The marginal
areas or 'prohibited areas' outside the pale of mainstream Brahmanical culture of gangetic valley
became accessible through various trade routes. The knowledge of the earliest routes comes from
the religious texts, which mention the travels of stray persons from place to place. With the
intensification of trade, especially with the Mediterranean world, the western texts mention a number
of cities and urban centres. Much information is also gathered from the archaeological evidences
testifying to long-distance exchange of goods. Thus, a broad, but indistinct picture of a network of

E
trade routes emerges. The most important among these was the 'Dakshinapatha', a route to south,
which opened up the areas south of Vindhyas. So important was this route that the whole country
OR
to the south came being designated 'Dakshinapatha'. A large number of articles, primarily raw
material of different type, were exported to the Mediterranean world, while a few were also imported.
The southern region comprising of Maharashtra, Andhra Pradesh, Tamil Nadu and Kerala, with
its long coastline actively participated in this trade mechanism. A number of large cities emerged on
strategic locations of trade routes and also as ports. Thus, the expanding trade definitely opened up
distant lands for the monks to venture out and preach.
SC

This process was accelerated and strengthened by emergence of powerful imperial states. During
this period emerged the Mauryan empire, the first major empire of the sub continent controlling
large geographic areas with differing polities and societies at various levels of social stratification in
its fold. The extent of Mauryan empire is known from the locations of Asokan edicts, which are
found as far south as Chitradurga district in Karnataka and Kurnool district in Andhra Pradesh. It
is postulated that subsequent emergence of the powerful state of Satavahanas and Ikshvakus in
Deccan, helped the spread of Buddhism in this region, which is marked by proliferation in Buddhist
GS

monastic sites during this period.


Buddhism came to be favoured by traders. Buddhism, with its opposition to the Brahmanical taboos
on purity and contamination, encouraged travel and in turn accelerated long distance trade. The
literary and archaeological records link Buddhism with king and the merchant. These sources portray
the social milieu of Buddhism as a complex urban environment with kings, wealthy merchants,
craftsmen and professionals. There is large number of references to urban centres in Buddhist literary
sources as opposed to stray references to rural settlements. The largest number of monks and nuns
of early sangha came from large towns and from powerful, wealthy families. There is a marked
preference to trade over other professions in the Buddhist literature. The donative inscriptions from
almost all Buddhist sites in central and southern India record donations primarily by traders, and
various craftsmen, occasionally from far off places.
Buddhism also provided much-needed support system to the changing cultural milieu. At the
ideological level, it influenced and encouraged the accumulation and reinvestment of wealth in
trading ventures by lay devotees, at the social level, donations to Buddhist monasteries provided
status to traders and other occupational groups, while at the economic level, the Buddhist
monasteries were repositories of information and essential skills such as writing. Moreover, the
organised institution of Buddhist sangha brought monasteries into closer contact with lay community
and provided identity and cohesiveness to trading groups.

Hints: History [15]

https://t.me/testseries4exams
Download All PDF form here :- https://t.me/pdf4exams

The association of trade, urbanism and powerful states with Buddhism is indicated by occurrence
of most of Buddhist sites of the period on strategic locations, either on trade route or near large
urban centre. Bharhut in central India occupied the northern end of the valley, in an area rich in
mineral resources. The sites in the Deccan were located on major trade routes. Junnar, with largest
cluster of caves was located at the head of Naneghat, an important pass. Similarly, Kanheri, another
important site, was located in the vicinity of port of Kalyan, similarly at Karla and at Kondane.
Amaravati and Nagarjunakonda in Andhra Pradesh were located near the flourishing capital cities
of Satavahanas and Ikshvakus. The other sites were located within rich, fertile, rice-growing Krishna
delta and along arterial routes.
Thus, Buddhism spread against the background of expanding trade network and the empire building
process of early historic period, both of which opened up routes to distant lands of southern India.
The well-organised institution of Buddhist sangha, the proselytizing efforts of dynamic monks and
the nature of Buddhism, which favoured trade and urban life-style, were some of the factors that
led to immense popularity of the sect during this period in central and southern India. A large
number of monasteries emerged on major trade routes and/or near large urban centres and thrived
on the large-scale donations, primarily by the trading community. When the trade dried up and

E
trade routes became inactive, the sect declined, though continued to survive in stray pockets till
very late. As the religion took a more organized form, especially once it obtained patronage from
OR
the imperium as well as the mercantile community, the religion also visually expressed it self through
the architecture.
6. (c) Evaluate the knowledge on medical science of Ancient Indian People.
The ancient Indian people were very profound and forerunner in science. They contributed in
different aspects of science like mathematics, astronomy, medical science, astrology e.t.c. But there
SC

achievements in medical science were much ahead from the time.


Archaic hindu medicine in its earliest context is to be found mostly in the hymns of Atharva Veda
and the vedic term bheshaja used to denote medicinal charms. It is a psychosomatic approach to
healing, part of a philosophical system, a scheme in which lay physician and the priest perform
their respective roles in controlling the ill of the body and the soul. Native pre- Aryan lore and
practiced were also absorbed. Many archeological evidences from Indus Valley Civilisation testify
the hygiene and medical practices.
GS

Note- apart from this charak, shushrut, dhanvantari must me mentioned here
7. (a) What were the impacts of Persian invasion in India? Is the impact could be noticed in
Ashokan pillars as well?
IMPACTS OF PERSIAN INVASION
• Introduction into India the Araminc form of writing, which later developed into the Kharoshthi
alphabet.
• Promotion of Indo-Iranian trade
• Geographical exploration of the Indus and the Arabian Sea, leading to opening of a new
water route.
• Fusion of Iranian/Persian features in the Mauryan art.
• Impact of Buddhism on the Zoroastrian religion of ancient Persia.
Some art historians have emphasized foreign influence, especially Persian influence, on the court
art of the Maurya Empire. It has been suggested that Ashoka got the idea of, inscribing proclamations
on pillars from the Achaemenids. It has been pointed out that the words dipi and lipi occur in the
inscriptions of Darius as well as Ashoka. The inscriptions of both kings begin in the third person
[16] Hints: History

https://t.me/testseries4exams
Download All PDF form here :- https://t.me/pdf4exams

and then move to the first person. Distinct Greek influence, and even greater Persian influence, has
been identified in the polished surface of the Ashokan pillars and the animal capitals. The stiff,
heraldic pose of the lions is seen as further evidence of Western influence. As far as this issue is
concerned, it has been pointed out many years ago that India formed part of an 'ancient east'-an
area extending from the Mediterranean Sea to the Ganga valley, which had some elements of a
common cultural heritage from very early times. There was plenty of interaction between ancient
India and ancient Iran, whether in the form of the conquest of Gandhara by the Persian emperors.
The use of the Aramaic script in certain Ashokan inscriptions in the north-west and the emergence
of Kharoshthi from this script were direct results of the interactions between India and West Asia.
At the same time, attention is also drawn to the many differences between the Maurya and Persian
pillars. The pillars of the Kumrahar hall do not have capitals, whereas those at Persepolis have
elaborate ones: The Persian pillars stand on bases, either shaped like a 'bell' (i.e., inverted lotus), or
a plain rectangular or circular block. In the Maurya pillars, on other hand, the inverted lotus appears
at the top of the shaft. The-shape and ornamentation of the Maurya lotus is different from the
Persian ones, the bulge typical of the former being absent in the latter. Most of the Persian pillars
have a fluted surface, while the Maurya pillars are smooth. The capitals of the Persian columns are

E
crowned with a duster of stylized palm leaves and. have two semi-bulls, lions, or unicorns seated
back to back, or an upright or inverted cup, with double volutes on the top. The Maurya-type
OR
abacus and independent animals carved in the round crowning the pillars are absent in the Persian
context. While there may be some similarities in specific features, the effect of the whole is completely
different. Moreover, by having pillars inscribed with his messages on dhamma, Ashoka transformed
them into epigraphic monuments of unique cultural meaning.
The issue of artistic 'influence' is a complex one. There is no doubt that artists and artistic motifs,
designs, and styles do travel-often over vast distances. In some instances, a particular motif is so
SC

strikingly similar to another that one seems to be inspired by the other cases, there is some stylistic
similarity, but also certain striking differences and innovations. Sometimes, similar designs can be
associated with different meanings. And at still other times, there is similarity in the choice of
motifs, but not in the style in which they are executed, pointing to the existence of a shared pool of
symbols that were considered to have meaning and significance in different cultures. If the Ashokan
pillars cannot in their entirety be attributed to Persian influence, they must have had an
undocumented prehistory within the subcontinent, perhaps a tradition of wooden carving but the
GS

transition from stone to wood was made in one magnificent leap, no doubt, spurred by the imperial
tastes and ambitions of the Maurya emperors.
7. (b) What were the major nastika philosophies in Ancient India and in what ways it
differed from astika philosophies. Write with special reference to Lokayat sect.
Indians distinguish two classes of Indian philosophies: astika and nastika. Theastika systems respect
the Vedas to some degree. They are: Sankhya, Yoga, Vedanta, Mimamsa, Nyaya, and Vaisheshika.
The nastika systems reject Vedic thought. They are: Jainism, Buddhism, and Lokayata.
Except for the Lokayata materialists, all these systems agreed about karma and reincarnation. Most
Indian systems also agreed on the doctrine of samsara: that today's humans have passed from birth
to birth from eternity. Except for Buddhism and Lokayata, the Indian systems agreed on the existence
of a permanent soul, or atman. In most systems, it was a kind of purification of the soul that leads
to moksha, though what this means varied from system to system.
Atheists and materialists were apparently common in ancient India, for the Hindu scriptures found
it necessary to respond to the arguments of non-believers on many occasions. The materialist systems
were often called "Lokayata," which means "that which is found among people in general."
Lokayata's skepticism about karma, reincarnation, and theology came from its epistemology.
Lokayata held that perception is the only valid source of knowledge, for all other sources like

Hints: History [17]

https://t.me/testseries4exams
Download All PDF form here :- https://t.me/pdf4exams

testimony and inference are unreliable. Perception revealed only the material world, made of the
four elements: air, fire, water, and earth. Minds and consciousness were, too, the products of matter.
Souls, gods, and the afterlife could not be perceived, and thus could not be said to exist. Religious
rituals were useless, and scriptures contained no special insight.
Thus, the only purpose of life was to enjoy pleasure and avoid pain. Critics described the ethics of
the Lokayata as egoistic, hedonistic, or even nihilistic.
But most Lokayata were naturalists. They believed things moved and transformed because of their
inherent natures, according to lawful necessity. Their fundamental principle was nature (svabhava).
The earliest known Indian materialist was Brhaspati, whose dates are unknown. He had no positive
system to advance, but merely denied orthodox views of theology, ethics, and dualism. He said:
"The whole Hindu system is a contrivance of the priesthood to secure a means of livelihood for
themselves." Because of him, earliest Indian materialism was sometimes called "Brhaspatya."
Another early materialist was Ajita Kesakambali (6th century B.C.), who lived as an ascetic despite
denying the afterlife, karma and morality. He was quoted as saying:

E
Ideas like generosity are the concepts of a stupid person. He who speaks of their existence, his
words are empty and confused; a cry of desperation.
Later Indian materialism is sometimes called Carvaka after the supposed author of the Barhaspatya
OR
sutras, which are now lost.
7. (c) "After the Kushanas, Mahayana Buddhism deviated a lot from the original Buddhism,
but even though it attracted the mass a lot." Critically analyse.
During the Kushanas the fourth Buddhist council was held at Kashmir, where Buddhism got divided
into two broad sects Hinyana and Mahayana. There were many major differences between Hinyana
SC

and Mahayana. Hinyana was much closer to original Buddhism whether Mahayana deviated a lot
from the original Buddhism.
Differences between Hinyana and Mahayana
• One of the differences is that Mahayana Buddhists believe that The Buddha (Siddhartha
Gautama) is a God but Hinayana Buddhists believe that The Buddha was an ordinary
Human
GS

• There is also another difference between Hinayana and Mahayana Buddhism this is that
Mahayana Buddhism followers think that it is better to help others reach Nirvana before you
do. But Hinayana Buddhism followers think that everyone needs to gain Nirvana by their
own
• Mahayana Buddhists all help others to reach Nirvana before they do because they think it
is better to help other people reach Nirvana than to reach Nirvana for yourself.
• However, Hinayana Buddhists think that reaching Nirvana is the ultimate goal of all Buddhists
and that you should gain Nirvana for yourself.
• One other difference between Mahayana and Hinayana Buddhism is that Mahayana
Buddhism followers think that all who trust in the Buddha will reach Nirvana. On other
hand, Hinayana Buddhism followers think that only Hinayana Buddhists will reach Nirvana.
Note- Discuss the popularity of Mahayana school
8. (a) Discuss the changing social pattern in post Mauryan and Gupta period with special
reference to verna and jati.
Our information on the Varna concept comes largely from the Dharmasastras, and description of it
[18] Hints: History

https://t.me/testseries4exams
Download All PDF form here :- https://t.me/pdf4exams

in those sources is not always corroborated, and occasionally even contradicted by other sources.
The concept of varna is seen to be closely associated with the concept of dharma, understood in
sense of universal law. Dharmasastras state that the society was made up of four orders, and later
a fifth order was added. The first four were the Brahman, Kshatriya, Vaisya and Sudra. The fifth
order was later identified with untouchables. This schema of social structure is traced from the
purushasukta, the tenth mandala of Rig Veda. This tenth mandala is believed to be a later addition.
The reference to Kshatriya, Vaisya and Sudra as a social category is only found in this last mandala
and is conspicuously absent in other parts of Rig-Veda.
In the later period, Manu provides a list of Varnas and occupations associated with them. Though
other Hindu sources recognise a hierarchical social composition of the contemporary society, yet
there is no sufficient evidence to suggest they resembled the varna system described by Manu. Had
the varna system functioned as a superimposed hierarchical layer of social groups, the distinction
between the four main groups and other permutation and combinations would have remained
very clear and distinct. What is curious however that while the identity of the brahmana and the
untouchable is generally clear, references to the intermediate groups often appear to be of a rather
confused, if not contradictory kind. In large number of instances provided in the non-Dharmasastra

E
sources one finds a contrary situation. Buddhism is viewed as a system, which was more sympathetic
to oppressed groups and it provided an economic, political, and social solution to the caste oppression.
OR
In Buddhist literature no one is ever described as belonging to brahmana varna, kshatriya varna,
vaisya or sudra varna. It seems to have remained a theoretical concept without any parallel in
actual practice. On the other hand, the terms jati and kula appear in concrete situations quite
frequently. What really seems to matter the Buddhists were the kula and jati divisions.
The vinaya pitaka states that there are two jatis: the low jati (hina jati) and the excellent jati (ukkatta
jati). Buddha also accepts this bipartite division, but at several instances refuted the relation of jati
SC

in the matter of spiritual attainment. In doing so, Buddha though recognises the importance of jati
and gotra in social interaction, but rules out their extrapolation to the spiritual field. In purana
kassapa, a distinct Buddhist text, six social divisions are conceptualised based on occupation, trade,
caste and sect affiliation. They are:
1) Kanhabi jati- Black jatis mostly comprising of those who follow a bloody trade, i.e., mutton
butchers, pork butchers, fowlers, hunters, fishermen, robbers etc. 2) Nilabhi jati- Blue jatis comprising
of Bhikshus. 3) Lohitabhi jati- Red jatis which includes Jainas. 4) Halladdabhi jati- Yellow jati,
GS

which includes white robbed householders or gahapatis. 5) Sukkabhi jati- White jati comprising of
Ajivikas and their followers.
The scheme not only provides this broad structure of the society in terms of different colour groups,
it further resolute low jati group into a hierarchical scheme of occupational groups. This textual
resolution of the low jatis into occupational groups starting with pukusuka should be taken to
indicate an order of lowness. This in overall character seems as a forerunner to Manu's scheme. The
Buddhists believed that good behaviour and wisdom being rewarded with rebirth in the high kulas
of Kshatriya and Brahmanas and Gahapatis. The opposite characteristics on the other hand would
result in rebirth in the low kulas of Chandala, nesadas, vennas and pukkusas.
Rhys Davids drawing conclusion from the recruitment practice and principle of Buddhism proves
that the jati was not a determining criterion in Buddhist Sangha. But, in practice the egalitarian
principle of Buddhism could not influence beyond life in Sangha. Other section of the society and
the social interaction however still followed the discriminatory practice of the caste system. Rhys
Davids believed that "had Buddha's view own the day, the evolution of social grades and distinctions
would have developed differently and the caste system would never have been built up". Oldenberg,
on the other hand, has pointed out that despite the fact that Buddhist theory acknowledged the
equal rights of all persons to be received into the Sangha, the actual composition of the Sangha
suggests that it was by no means in the keeping with the theory of equality and that a marked
Hints: History [19]

https://t.me/testseries4exams
Download All PDF form here :- https://t.me/pdf4exams

leaning to aristocracy seems to have lingered in ancient Buddhism. Similarly, Fick states that the
development of caste was in no way broken or even retarded by Buddhism. Charles Eliot in his
book Hinduism and Budhism also suggest that while Buddha attacked both the ritual and philosophy
of the brahmans, so that after his time the sacrificial system never regained its earlier prestige, he
was less effective as a social reformer. Buddhism did oppose the Brahministic ritualism, but did not
preach against the caste system as whole. E. Senart in his book 'Caste in India' also writes that the
conflict between the Buddhists and Brahmans was primarily a struggle for influence, and that
there was nothing in the Buddhists stand which aimed at changing the entire caste system.
Some of the historical evidence in the Gupta Age points to departures from the earlier Smriti law on
varna and jati. An inscription of 5th century AD refers to two kshatriya merchants living in a city
in the upper-Ganga basin, while another inscription of the same century mentions of a body of
weavers from Gujarat as having gradually adopted various other occupations in their new home in
Malwa. This social dynamism is proved by a number of authentic instances of brahmanas and
Kshatriyas adopting the occupations of the classes below them, and of Vaisyas and Sudras following
those of the classes above them. This social dynamism needs to be understood in the economic and
political dynamism of this period. The economic expansion was integrally related to the social

E
integration process. The village economy grew from subsistence production stage to produce social
surplus to support trade and commerce. The imperial polity integrated vast regions into a single
OR
political unit allowing different people, skills and resources to interact. The land grant to kshatriyas
expanded the agricultural practice to nook and corner of the country. The spread of Brahman
groups stretched brahmanical nuclei to foster systematic acculturation in such regions to the Sanskritic
mould. The social groups enjoying different grades of social status were integrated into the economy
and regional polity. The emergence of small kingdoms at the end of Gupta period thus created
many groups to claim Kshatriya status. The economic opportunities lured some brahmanas to take
SC

up trade. New technology and craft activities provided new opportunities to Vaisyas and sudras.
These opportunities of economic interaction created new rules of social regulations. As we have
discussed in the earlier section, the rules of inter-marriage became more rigid. Though, some examples
of intermarriages between varnas (both anuloma and pratiloma) can be inferred from the literary
sources, yet they seem to be confined to the social and economic elites. On the whole, the hierarchical
model of varna system could not be rigidly enforced in practice, since it would require a static
society for proper functioning.
GS

8. (b) "The Gupta period could be called a golden age for the development in astronomy and
mathematics." Examine.
The developments in various spheres of the natural sciences need to be investigated and subjected
to sober analysis, avoiding exaggerated claims but acknowledging important contributions and
breakthroughs. The earliest evidence of ancient Indian astronomical knowledge is contained in the
Vedanga texts on jyotisha or astrology, the main focus of which was to fix the date of sacrificial
rituals. The Sanskrit names of the signs of the zodiac have Greek origins, and it seems that Greek
influence led to the sequence of planets being fixed in the names of the seven days of the week in
Indian texts. A Sanskrit text known as the Yavanajataka reflects the transmission of Hellenistic
astronomical ideas into India. However, Indian astronomers appear to have made certain major
breakthroughs independently. Varahamihira's Panchasiddhantika (6th century) summarizes the
astronomical works and ideas of the preceding centuries, but ascribes their authorship to divine or
semi-divine beings.
The earliest known historical astronomer in India is Aryabhata I, who wrote at least two works- the
Aryabhatiya, a text which survives and deals with astronomy and mathematics, and the Aryabhata-
siddhanta, which is known only through references in later works. This astronomer seems to have
been a native of Ashmaka country (on the Godavari). This is clear from the fact that the 7th century
commentator Bhaskara I calls the Aryabhatiya the Ashmakatantra and Ashmakiya, and the
[20] Hints: History

https://t.me/testseries4exams
Download All PDF form here :- https://t.me/pdf4exams

followers of Aryabhata Ashmakiyas. A statement in the Aryabhatiya indicates that Aryabhata


lived in Kusumapura, i.e., Pataliputra. He was obviously aware of the ideas and methods of his
predecessors, but struck his own course. 'I dived deep in the ocean of astronomical theories, true
and false,' he writes, 'and rescued the precious sunken jewel of true knowledge by means of the
boat of my own intellect.'
Aryabhata had an earth-centric view of the universe-he thought that the planets moved around
the earth in circular epicycles. Nonetheless, he was the first astronomer to give a scientific explanation
of eclipses. He established that eclipses were not caused by the demons Rahu and Ketu, but by the
moon coming within the earth's shadow or between the earth and sun. He worked out how to
ascertain which part of the moon would be obscured during an eclipse. He was also the first to
discover that the earth rotated on its axis. Another one of his many achievements was to find out
the sine functions and use them in astronomy. He worked out the correct equation for calculating
the orbit of a planet, and gave an extremely accurate estimate of the length of a year
(365.2586805days). Unfortunately we do not know about the experiments or methods used by
Aryabhata in reaching such momentous conclusions.

E
Varahamihira was a 6th century astrologer, astronomer, and mathematician who belonged to Avanti
(in western Malwa). Mention has already been made of his Panchasiddhantika, wherein he
OR
summarized the five astronomical schools prevalent in the time. His Brihatsamhita is an
encyclopaedic work dealing with diverse topics including how to sharpen swords, how to ascertain
the value of precious metals and stones, how to make trees bear fruit out of season, how to distinguish
the good breeds of animals, and how to divine the location of water. It also discusses the nature and
structure of temples, palaces, and houses. It gives an explanation of seasons and discusses
meteorological issues such as the correlation between the clouds, winds, and amount of rainfall.
SC

Brahmagupta, an astronomer and mathematician of the late 6th/7th century, was the author of
the Brahmasputasiddhanta (628 CE) and the Khandakhadyaka (665 CE). These texts became very
influential within India, and their Arab translations and adaptations introduced Indian astronomy
to the Arabs. The Brahmasputasiddhanta is also the first surviving Indian text containing a systematic
discussion of astronomical instruments, as well as methods of computing astronomical elements
from readings taken with them (Sarma, 1986). The instruments include accessories, astronomical
instruments for measuring time and observing the celestial bodies, instruments that turn automatically
GS

for the duration of one day, and ones that rotate perpetually. The accessories (samsadhana) comprise
water, a pair of compasses (bhrama), plumb-line (avalamba), hypotenuse (karna), shadow (chhaya),
mid-day (dinardha), the sun, and the local latitude (aksha). The text mentions nine astronomical
instruments-chakra (a circular wooden plate graduated into 360º), dhanus (a semicircular plate),
turyagola (a quarter plate), yashti (staff), shanku (gnomon), ghatika (clepsydra), kapala (a
horizontally placed circular plate), kartari (two semi-circular plates joined together at different
levels), and pitha (a horizontally placed chakra). S. R. Sarma points out that the instruments, made
of wood or bamboo, are very simple in design and could not have provided much precision in
measurement. This suggests that astronomers probably relied more on their superior computing
skills. However, Brahmagupta also referred to complex automatic devices called svayamvaha yantras,
which reflects an awareness of the idea of perpetual motion.
The roots of Indian mathematics can be traced to the Shulvasutras, appendices to the Shrautasutras
(Hayashi, 2003). Shulva means measurement and the Shulvasutras are manuals for the preparation
of the site where Vedic sacrificial rituals were to be performed, dealing especially with the construction
of Vedic brick fire altars. Among other things, these manuals contain one of the earliest expressions
of the principle behind what later came to be known as Pythagora's theorem in geometry (the
ancient Babylonians were also aware of this principle). The Shulvasutras also made suggestions for
squaring a circle, i.e., to construct, using only ruler and compasses, a square whose area is equal to
that of a given circle.
Hints: History [21]

https://t.me/testseries4exams
Download All PDF form here :- https://t.me/pdf4exams

In later times, the term ganita-shastra was the most frequently used term for mathematical science.
One of the most important discoveries of ancient Indian mathematicians was the decimal system of
notation, based on the place value of the first nine numbers and the use of a symbol known as bindu
for zero (see Sarma, 1988; Baig and Sarma, 2003). The use of this system greatly simplified arithmetical
calculations. The oldest datable evidence of the decimal place-value system of notation is in a 3rd
century work on astrology called the Yavanajataka by Sphujidhvaja (Hayashi, 2003: 366). This
work does not, however, mention the zero. The zero symbol, a dot, was used in metrics (chhandas)
by Pingala in the Chhandasutra, a pre-2nd century BCE work. Varahamihira's Panchasiddhantika
is the earliest dateable text to give zero both as a symbol and as a number.
The decimal system of notation was used by Varahamihira and was referred to by Aryabhata in his
Aryabhatiya. Aryabhata's method of extracting the square root and cube root presupposes the
decimal place value of numbers. This shows that Indian mathematicians were using the system in
the 5th century CE. In Europe, the old cumbersome system was followed till the 12th century,
when the Europeans learnt the new system from the Arabs. Arab writers such as Ibn Washiya, Al-
Masudi, and Al-Biruni in fact give the credit for the discovery of the system to the 'Hindus'.

E
Aryabhata's Aryabhatiya is a work on astronomy and deals only incidentally with problems of
mathematics. Along with rules of involution and evolution, it deals with the arithmetical progression
of numbers and their squares and cubes. In the field of geometry, Aryabhata describes the various
OR
properties of a circle and gives a very accurate value for pi (?) correct to 4 decimal places at 3.1416.
Aryabhata is regarded as the father of algebra. His work solves a number of complex simultaneous
equations. The use of the sine functions in solving problems in astronomy indicates the development
of trigonometry. The Aryabhatiya gives tables for the trigonometric ratio sine (called jya in Sanskrit)
for angles from 0 to 90 degrees at intervals of 3¾ degrees. The same sine tables are also found in the
Surya Siddhanta. Aryabhata also perfected the methods of solving in integers certain types of
SC

indeterminate equations. Later mathematicians such as Brahmagupta and Bhaskara II also made
contributions in this sphere. Unlike Greek writers on geometry, ancient Indian mathematicians did
not give proofs or demonstrations.
Reference may be made here to some of the developments in later centuries as well. In the 7th
century, Indian mathematics came to be divided into two main areas-arithmetic with mensuration
and algebra. Bhaskara I (early 7th century) wrote a commentary on the Aryabhatiya, where he
GS

gave an interesting geometrical treatment for algebraic formulae. Brahmagupta (7th century) made
important contributions to geometry. He was the first mathematician to discuss the method of
obtaining a cyclic quadrilateral having rational sides and to give the area of a cyclic quadrilateral.
He also put forward theories on the circum-diameter of a triangle and for finding the diagonals of
a cyclic quadrilateral in terms of its sides. Mahavira (9th century) was a famous mathematician of
Karnataka who lived in the court of the Rashtrakuta king Amoghavarsha Nripatunga of
Manyakheta. He wrote a book called Ganitasarasangraha which dealt with various mathematical
problems. He also gave formulae for the area and circumference of an ellipse. The formula he gave
for the area of an ellipse was incorrect, but the one for the circumference was correct. Bhaskara II
(12th century), author of the Lilavati was another important mathematician, whose writings contain
some important ideas of calculus.
8. (c) Over the time it became difficult for the individual to upgrade in the social order but
social mobilization was always evident in Indian society. Critically Examine.
Social mobility was always present in India but its nature kept on changing. In Early Vedic period
verna system was not that too rigid so it was a common and not very surprising but when in later
Vedic period verna rules became hard and fast than it was always expected from the Kshatriya
verna to stop such practices. Even though it can not be said that the practice of changing the verna
continued till date. Earlier definitely references were there for individual mobility but later it was
possible only for the communities.
[22] Hints: History

https://t.me/testseries4exams
Download All PDF form here :- https://t.me/pdf4exams

Early legend tells of Visvamitra, a ksatriya who, by penance and piety, became a brahman and a
seer (rsi) to boot; but as time went on such raising of one's rank in the social scale became more and
more difficult, and finally virtually impossible, though convenient fictions sometimes permitted
kings and chiefs of low status to find legendary ksatriya ancestors and advance in the class hierarchy.
While it became difficult for the individual to rise, it grew progressively more easy for him to fall.
Every breach of the manifold regulations of one's class entailed impurity and outcasting, either
permanent or temporary. The law books give long lists of penances for the restoration of the
unfortunate offender, ranging from trivial ones, such as bathing or touching Ganga water, to others
so rigorous, that they must usually have resulted in the death of the penitent. Secular literature,
however, tells many stories of high-class people infringing the rules of purity without doing penance,
and no doubt: the sophisticated townsman often too oks his class responsibilities lightly,
The continual injunctions to the king to ensure that "confusion of class" (Varna-samkara) did not
take place indicate that such confusion was an ever-present danger in the mind of the orthodox
brahman. The class system was indeed a very fragile thing. In the golden age the classes were
stable; but the legendary king Vena , among his many other crimes, had encouraged miscegenation,
and from this beginning confusion of class had increased, and was a special feature of the Kali-

E
Yuga, the last degenerate age of the aeon, which was fast nearing its close. The good king, therefore,
was advised to spare no effort to maintain the purity of the classes, and many dynasties took special
OR
pride in their efforts in this direction Before the tightening of the social system in the Middle Ages
confusion of class was comparatively frequent and some form of interclass marriage were expressly
permitted The type of marriage known to anthropologists as hypergamous, when the husband is
of higher class than the wife, was by no means disapproved of; on the other hand hypogamous
marriage, when the wife's status was higher than that of the husband was always frowned on. The
former was In accordance with the direction of the hair" (anuloma) smooth and natural, while the
SC

latter was "against the hair'', or "brushing the wrong way" (pratiloma). This distinction is to be
found in other societies; for instance in Victorian England the peer who married an actress rarely
incurred the same scorn and ostracism as the lady who married her groom.
The earlier legal literature permitted anuloma or hypergamous marriage, provided that a man's
first wife was of his own class. Generally brahmans were forbidden to take sudra wives, but one
lawbook allowed even this, and Bana, the 7th-century poet, who was a brahman, had a stepbrother
by a sudra mother. Various mixed classes, many of them the forerunners of later castes, were said
GS

to be the products of marriages of this type, and their members were not looked on as in any way
unclean, but enjoyed a position intermediate between that of the two parents. Of the groups thought
to have descended from hypergamous marriage only the nisada, in theory a cross between a brahman
and a sudra woman, was thought to be impure.
Hypogamous or pratiloma marriage, on the-other hand, produced, offspring whose status was
lower than that of either parent. Thus the candalas were believed to have descended from marriages
between sudras and brahman women. The only exceptions were the class of charioteers, or sutas,
thought to have sprung from the hypo gamy of ksatriyas and brahmans, and the bards or magadhas,
descended from Vaisya fathers and ksatriya mothers, both of whom were a well respected. The
complex system of sub-classes low in the social scale, out of which the Indian caste system developed,
was believed to be wholly the result of "confusion of class". This tradition was accepted by early
Indologists, but, as we shall see, is completely un-founded.



Hints: History [23]

https://t.me/testseries4exams

You might also like